How many terms are required for harmonic series of degrees to “cover” a full $360^text{o}$ circle?












2












$begingroup$


This question accidentally came to my mind when reading about harmonic series. I've never been able to find an answer on the Internet. Consider $H_n$ which is the $n$-th harmonic number:
$$
H_n = 1 + {1over 2} + {1over 3} + cdots + {1over n}
$$



Not sure whether it's valid but suppose we have a circle which's full rotation is known to be $360^text{o}$. Lets define the following series:
$$
S_n^text{o} = (1)^text{o} + left({1over 2}right)^text{o} + left({1over 3}right)^text{o} + cdots
$$



Since harmonic series is divergent and its sum tends to infinity, then at some point we should have "covered" the whole circumference. If we now define an "$n$-th harmonic degree number" as $H_n^text{o}$. Then we have to solve the following inequality for $n$:
$$
H_n^text{o} ge 360^text{o}
$$



Please note that i'm not very familiar with series and only have basic calculus knowledge like limits, derivatives and Taylor expansion. Also I may have misused a lot of terms in this question, so please comment in order for me to improve it. Apart from that I'm basically interested in two things:





  1. Is it valid to consider the harmonic sum of degrees rather than the sum of rationals?

  2. If so what would be the way to find the index of $H_n^text{o}$ such that the whole circle is "covered"?











share|cite|improve this question











$endgroup$

















    2












    $begingroup$


    This question accidentally came to my mind when reading about harmonic series. I've never been able to find an answer on the Internet. Consider $H_n$ which is the $n$-th harmonic number:
    $$
    H_n = 1 + {1over 2} + {1over 3} + cdots + {1over n}
    $$



    Not sure whether it's valid but suppose we have a circle which's full rotation is known to be $360^text{o}$. Lets define the following series:
    $$
    S_n^text{o} = (1)^text{o} + left({1over 2}right)^text{o} + left({1over 3}right)^text{o} + cdots
    $$



    Since harmonic series is divergent and its sum tends to infinity, then at some point we should have "covered" the whole circumference. If we now define an "$n$-th harmonic degree number" as $H_n^text{o}$. Then we have to solve the following inequality for $n$:
    $$
    H_n^text{o} ge 360^text{o}
    $$



    Please note that i'm not very familiar with series and only have basic calculus knowledge like limits, derivatives and Taylor expansion. Also I may have misused a lot of terms in this question, so please comment in order for me to improve it. Apart from that I'm basically interested in two things:





    1. Is it valid to consider the harmonic sum of degrees rather than the sum of rationals?

    2. If so what would be the way to find the index of $H_n^text{o}$ such that the whole circle is "covered"?











    share|cite|improve this question











    $endgroup$















      2












      2








      2





      $begingroup$


      This question accidentally came to my mind when reading about harmonic series. I've never been able to find an answer on the Internet. Consider $H_n$ which is the $n$-th harmonic number:
      $$
      H_n = 1 + {1over 2} + {1over 3} + cdots + {1over n}
      $$



      Not sure whether it's valid but suppose we have a circle which's full rotation is known to be $360^text{o}$. Lets define the following series:
      $$
      S_n^text{o} = (1)^text{o} + left({1over 2}right)^text{o} + left({1over 3}right)^text{o} + cdots
      $$



      Since harmonic series is divergent and its sum tends to infinity, then at some point we should have "covered" the whole circumference. If we now define an "$n$-th harmonic degree number" as $H_n^text{o}$. Then we have to solve the following inequality for $n$:
      $$
      H_n^text{o} ge 360^text{o}
      $$



      Please note that i'm not very familiar with series and only have basic calculus knowledge like limits, derivatives and Taylor expansion. Also I may have misused a lot of terms in this question, so please comment in order for me to improve it. Apart from that I'm basically interested in two things:





      1. Is it valid to consider the harmonic sum of degrees rather than the sum of rationals?

      2. If so what would be the way to find the index of $H_n^text{o}$ such that the whole circle is "covered"?











      share|cite|improve this question











      $endgroup$




      This question accidentally came to my mind when reading about harmonic series. I've never been able to find an answer on the Internet. Consider $H_n$ which is the $n$-th harmonic number:
      $$
      H_n = 1 + {1over 2} + {1over 3} + cdots + {1over n}
      $$



      Not sure whether it's valid but suppose we have a circle which's full rotation is known to be $360^text{o}$. Lets define the following series:
      $$
      S_n^text{o} = (1)^text{o} + left({1over 2}right)^text{o} + left({1over 3}right)^text{o} + cdots
      $$



      Since harmonic series is divergent and its sum tends to infinity, then at some point we should have "covered" the whole circumference. If we now define an "$n$-th harmonic degree number" as $H_n^text{o}$. Then we have to solve the following inequality for $n$:
      $$
      H_n^text{o} ge 360^text{o}
      $$



      Please note that i'm not very familiar with series and only have basic calculus knowledge like limits, derivatives and Taylor expansion. Also I may have misused a lot of terms in this question, so please comment in order for me to improve it. Apart from that I'm basically interested in two things:





      1. Is it valid to consider the harmonic sum of degrees rather than the sum of rationals?

      2. If so what would be the way to find the index of $H_n^text{o}$ such that the whole circle is "covered"?








      calculus sequences-and-series summation harmonic-numbers






      share|cite|improve this question















      share|cite|improve this question













      share|cite|improve this question




      share|cite|improve this question








      edited Jan 22 at 14:50









      José Carlos Santos

      164k22131234




      164k22131234










      asked Jan 22 at 14:35









      romanroman

      2,30921224




      2,30921224






















          2 Answers
          2






          active

          oldest

          votes


















          3












          $begingroup$

          Since $H_n$ is close to $log n$ if $n$ is large enough, you will have to take $n$ about $e^{360-gamma}$, which is about $1.25times10^{156}$. Here, $gamma$ is the Euler-Mascheroni constant.






          share|cite|improve this answer











          $endgroup$













          • $begingroup$
            Indeed. Do you mind if I edit my answer, adding to it your suggestion?
            $endgroup$
            – José Carlos Santos
            Jan 22 at 14:49










          • $begingroup$
            whoa, that's a long way round. Thanks!
            $endgroup$
            – roman
            Jan 22 at 14:53



















          3












          $begingroup$

          Note that for large $n$, $H_nsim ln n$, so $ln napprox 360$ means that $napprox 2.2times 10^{156}$.






          share|cite|improve this answer











          $endgroup$









          • 1




            $begingroup$
            It is misleading to give so many significant figures! The approximation $H_nsim ln n$ is not nearly so precise.
            $endgroup$
            – TonyK
            Jan 22 at 14:51











          Your Answer





          StackExchange.ifUsing("editor", function () {
          return StackExchange.using("mathjaxEditing", function () {
          StackExchange.MarkdownEditor.creationCallbacks.add(function (editor, postfix) {
          StackExchange.mathjaxEditing.prepareWmdForMathJax(editor, postfix, [["$", "$"], ["\\(","\\)"]]);
          });
          });
          }, "mathjax-editing");

          StackExchange.ready(function() {
          var channelOptions = {
          tags: "".split(" "),
          id: "69"
          };
          initTagRenderer("".split(" "), "".split(" "), channelOptions);

          StackExchange.using("externalEditor", function() {
          // Have to fire editor after snippets, if snippets enabled
          if (StackExchange.settings.snippets.snippetsEnabled) {
          StackExchange.using("snippets", function() {
          createEditor();
          });
          }
          else {
          createEditor();
          }
          });

          function createEditor() {
          StackExchange.prepareEditor({
          heartbeatType: 'answer',
          autoActivateHeartbeat: false,
          convertImagesToLinks: true,
          noModals: true,
          showLowRepImageUploadWarning: true,
          reputationToPostImages: 10,
          bindNavPrevention: true,
          postfix: "",
          imageUploader: {
          brandingHtml: "Powered by u003ca class="icon-imgur-white" href="https://imgur.com/"u003eu003c/au003e",
          contentPolicyHtml: "User contributions licensed under u003ca href="https://creativecommons.org/licenses/by-sa/3.0/"u003ecc by-sa 3.0 with attribution requiredu003c/au003e u003ca href="https://stackoverflow.com/legal/content-policy"u003e(content policy)u003c/au003e",
          allowUrls: true
          },
          noCode: true, onDemand: true,
          discardSelector: ".discard-answer"
          ,immediatelyShowMarkdownHelp:true
          });


          }
          });














          draft saved

          draft discarded


















          StackExchange.ready(
          function () {
          StackExchange.openid.initPostLogin('.new-post-login', 'https%3a%2f%2fmath.stackexchange.com%2fquestions%2f3083244%2fhow-many-terms-are-required-for-harmonic-series-of-degrees-to-cover-a-full-36%23new-answer', 'question_page');
          }
          );

          Post as a guest















          Required, but never shown

























          2 Answers
          2






          active

          oldest

          votes








          2 Answers
          2






          active

          oldest

          votes









          active

          oldest

          votes






          active

          oldest

          votes









          3












          $begingroup$

          Since $H_n$ is close to $log n$ if $n$ is large enough, you will have to take $n$ about $e^{360-gamma}$, which is about $1.25times10^{156}$. Here, $gamma$ is the Euler-Mascheroni constant.






          share|cite|improve this answer











          $endgroup$













          • $begingroup$
            Indeed. Do you mind if I edit my answer, adding to it your suggestion?
            $endgroup$
            – José Carlos Santos
            Jan 22 at 14:49










          • $begingroup$
            whoa, that's a long way round. Thanks!
            $endgroup$
            – roman
            Jan 22 at 14:53
















          3












          $begingroup$

          Since $H_n$ is close to $log n$ if $n$ is large enough, you will have to take $n$ about $e^{360-gamma}$, which is about $1.25times10^{156}$. Here, $gamma$ is the Euler-Mascheroni constant.






          share|cite|improve this answer











          $endgroup$













          • $begingroup$
            Indeed. Do you mind if I edit my answer, adding to it your suggestion?
            $endgroup$
            – José Carlos Santos
            Jan 22 at 14:49










          • $begingroup$
            whoa, that's a long way round. Thanks!
            $endgroup$
            – roman
            Jan 22 at 14:53














          3












          3








          3





          $begingroup$

          Since $H_n$ is close to $log n$ if $n$ is large enough, you will have to take $n$ about $e^{360-gamma}$, which is about $1.25times10^{156}$. Here, $gamma$ is the Euler-Mascheroni constant.






          share|cite|improve this answer











          $endgroup$



          Since $H_n$ is close to $log n$ if $n$ is large enough, you will have to take $n$ about $e^{360-gamma}$, which is about $1.25times10^{156}$. Here, $gamma$ is the Euler-Mascheroni constant.







          share|cite|improve this answer














          share|cite|improve this answer



          share|cite|improve this answer








          edited Jan 22 at 14:53

























          answered Jan 22 at 14:44









          José Carlos SantosJosé Carlos Santos

          164k22131234




          164k22131234












          • $begingroup$
            Indeed. Do you mind if I edit my answer, adding to it your suggestion?
            $endgroup$
            – José Carlos Santos
            Jan 22 at 14:49










          • $begingroup$
            whoa, that's a long way round. Thanks!
            $endgroup$
            – roman
            Jan 22 at 14:53


















          • $begingroup$
            Indeed. Do you mind if I edit my answer, adding to it your suggestion?
            $endgroup$
            – José Carlos Santos
            Jan 22 at 14:49










          • $begingroup$
            whoa, that's a long way round. Thanks!
            $endgroup$
            – roman
            Jan 22 at 14:53
















          $begingroup$
          Indeed. Do you mind if I edit my answer, adding to it your suggestion?
          $endgroup$
          – José Carlos Santos
          Jan 22 at 14:49




          $begingroup$
          Indeed. Do you mind if I edit my answer, adding to it your suggestion?
          $endgroup$
          – José Carlos Santos
          Jan 22 at 14:49












          $begingroup$
          whoa, that's a long way round. Thanks!
          $endgroup$
          – roman
          Jan 22 at 14:53




          $begingroup$
          whoa, that's a long way round. Thanks!
          $endgroup$
          – roman
          Jan 22 at 14:53











          3












          $begingroup$

          Note that for large $n$, $H_nsim ln n$, so $ln napprox 360$ means that $napprox 2.2times 10^{156}$.






          share|cite|improve this answer











          $endgroup$









          • 1




            $begingroup$
            It is misleading to give so many significant figures! The approximation $H_nsim ln n$ is not nearly so precise.
            $endgroup$
            – TonyK
            Jan 22 at 14:51
















          3












          $begingroup$

          Note that for large $n$, $H_nsim ln n$, so $ln napprox 360$ means that $napprox 2.2times 10^{156}$.






          share|cite|improve this answer











          $endgroup$









          • 1




            $begingroup$
            It is misleading to give so many significant figures! The approximation $H_nsim ln n$ is not nearly so precise.
            $endgroup$
            – TonyK
            Jan 22 at 14:51














          3












          3








          3





          $begingroup$

          Note that for large $n$, $H_nsim ln n$, so $ln napprox 360$ means that $napprox 2.2times 10^{156}$.






          share|cite|improve this answer











          $endgroup$



          Note that for large $n$, $H_nsim ln n$, so $ln napprox 360$ means that $napprox 2.2times 10^{156}$.







          share|cite|improve this answer














          share|cite|improve this answer



          share|cite|improve this answer








          edited Jan 22 at 14:56

























          answered Jan 22 at 14:44









          rogerlrogerl

          18k22747




          18k22747








          • 1




            $begingroup$
            It is misleading to give so many significant figures! The approximation $H_nsim ln n$ is not nearly so precise.
            $endgroup$
            – TonyK
            Jan 22 at 14:51














          • 1




            $begingroup$
            It is misleading to give so many significant figures! The approximation $H_nsim ln n$ is not nearly so precise.
            $endgroup$
            – TonyK
            Jan 22 at 14:51








          1




          1




          $begingroup$
          It is misleading to give so many significant figures! The approximation $H_nsim ln n$ is not nearly so precise.
          $endgroup$
          – TonyK
          Jan 22 at 14:51




          $begingroup$
          It is misleading to give so many significant figures! The approximation $H_nsim ln n$ is not nearly so precise.
          $endgroup$
          – TonyK
          Jan 22 at 14:51


















          draft saved

          draft discarded




















































          Thanks for contributing an answer to Mathematics Stack Exchange!


          • Please be sure to answer the question. Provide details and share your research!

          But avoid



          • Asking for help, clarification, or responding to other answers.

          • Making statements based on opinion; back them up with references or personal experience.


          Use MathJax to format equations. MathJax reference.


          To learn more, see our tips on writing great answers.




          draft saved


          draft discarded














          StackExchange.ready(
          function () {
          StackExchange.openid.initPostLogin('.new-post-login', 'https%3a%2f%2fmath.stackexchange.com%2fquestions%2f3083244%2fhow-many-terms-are-required-for-harmonic-series-of-degrees-to-cover-a-full-36%23new-answer', 'question_page');
          }
          );

          Post as a guest















          Required, but never shown





















































          Required, but never shown














          Required, but never shown












          Required, but never shown







          Required, but never shown

































          Required, but never shown














          Required, but never shown












          Required, but never shown







          Required, but never shown







          Popular posts from this blog

          Mario Kart Wii

          What does “Dominus providebit” mean?

          Antonio Litta Visconti Arese